LSAT and Law School Admissions Forum

Get expert LSAT preparation and law school admissions advice from PowerScore Test Preparation.

 Administrator
PowerScore Staff
  • PowerScore Staff
  • Posts: 8917
  • Joined: Feb 02, 2011
|
#81296
Complete Question Explanation

The correct answer choice is (A).

Answer choice (A): This is the correct answer choice.

Answer choice (B):

Answer choice (C):

Answer choice (D):

Answer choice (E):

This explanation is still in progress. Please post any questions below!
 mpoulson
  • Posts: 148
  • Joined: Mar 25, 2016
|
#25961
Hello,

I was trying to determine why the answer to this question was A and not E. A initially seems to contradict the author's point of saying egregious penalties could harm a lot of workers by shutting down the company. Upon further review, E doesn't seem to fit with the author's main point, but I still need help understanding why A is correct. Thank you.

V/r,

Micah
 Ladan Soleimani
PowerScore Staff
  • PowerScore Staff
  • Posts: 43
  • Joined: Oct 08, 2015
|
#25987
Hi Micah,

The key with this question is that the question stem is asking about the view of the economists referred to in the passage, not the author's viewpoint. The economists believe that the only thing that matters is a cost benefit ration of the crime; "the penalty levied should exceed the profit that accrued to the corporation as a result of committing the crime". So if the benefit is $6 million then the fine would need to be $7 billion; it wouldn't matter to them if that $7 million put the corporation out of business, making answer choice (A) correct.

Ladan
 chian9010
  • Posts: 81
  • Joined: Jun 08, 2018
|
#50040
Hi,

I understand why A is the correct answer. However, could anyone please illustrate why B is incorrect?
 Who Ray
PowerScore Staff
  • PowerScore Staff
  • Posts: 27
  • Joined: Jul 31, 2018
|
#50437
Hey there Chian!

If you look at the last few lines, the author says: "Thus, some other criterion in addition to the reckoning of cost and benefit—such as the assignment of moral weight to(50) particular crimes—is necessary so that penalties for corporate crimes will be practical as well as just."
This shows that the author disagrees with the economists that only economic forces should matter for determining punishments, and the author includes morals as a possible factor that could matter.

If you were tempted by B because it brought up the community's morals, the last few lines do not give all of the possible factors that could matter, so you cannot use that section to eliminate this answer choice. Also, you can look around lines 13-17, ("In arguing thus, the economists hold that the fact that a community may find some crimes more abhorrent than others or wish to send a message about(15) the importance of some values—such as, say, not endangering citizens’ health by selling tainted food—should not be a factor in determining penalties.") and see that the community's morals are a potential factor in evaluating the proper punishment for a crime.
Therefore, it seems likely that the author would disagree with AC B.

Hope that helps!
Who Ray
 ataraxia10
  • Posts: 46
  • Joined: Oct 04, 2018
|
#66540
Can anyone tell me if E is the wrong answer because it's irrelevant or because it belongs to the author's view? I chose E but from further analysis of E, I determined it is neither directly the author's nor the economists' point of view.
It is not the author's because the author thinks that "estimated detection ratio" needs to be taken into account, not simply the "number of times the corporation has previously been convicted of the crime."
We cannot attribute this to the economists' view because there is no mention in the passage of whether or not the economists will agree with this statement. The passage only dictates that the economists want the penalty to be larger than the profit accrued as a result of the crime. Essentially, the only factor that the economists think we should consider is the size of the penalty in comparison to the profit, which points to A as the likeliest correct choice.
User avatar
 KelseyWoods
PowerScore Staff
  • PowerScore Staff
  • Posts: 1079
  • Joined: Jun 26, 2013
|
#66825
Hi Ataraxia!

I agree with your analysis. Answer choice (E) does not reflect the economist's point of view because they only want the penalty to exceed the profit. It also does not reflect the author's point of view as the estimated detection ratio is not just about the number of times the corporation has been previously convicted of the crime but rather the estimation of how often those types of crimes are detected in general.

Great job!

Best,
Kelsey
 lanereuden
  • Posts: 147
  • Joined: May 30, 2019
|
#67536
Okay so I thought the answer here was D

Economists’ naive view: Penalty higher than profits.
Economists’ reformed view: Penalty higher than profits, accounting for detection ratios. (Higher penalty)
Author’s view: Penalty in between first two, with adjustment for community moral standards.

Since I thought like this , I couldn’t tell if 13 was asking from naive or reformed point of view.

Thus I picked D, since reformed economists believe that likelihood of recommitting crime (detection rate) should be accounted for.
Please explain where I went wrong.
Also, A is strange because the economists and their stance on w/n it goes out of business is not explicit. And so, only by implication is supported. How is it that this implication, as you, Ladan, said with 7 greater than 6 mil example, sufficient evidence for this question?
 dyogenes
  • Posts: 7
  • Joined: Mar 11, 2019
|
#67668
Lanereuden,

I went wrong in the exact same place, and after looking it over am still confused. I would structure the interaction something like this:
It reads like the economists only have one view: that the penalty should be informed only by the full/complete reckoning of cost/benefit of committing the crime. The argument proceeds by saying that after full consideration of what a complete reckoning would mean, the view is untenable due to the impracticality of heavy costs that can be fatal to the business and its employees' jobs.

The passage then affirms that the implementation of the approach endorsed by the economists requires that detection ratios be high enough for courts to just ignore them. Doesn't this seem to imply that the economists are at least noncommittal and at most against situations where detection ratios are high enough that courts can't ignore them?

The passage implies that the economists are not fully aware of all the repercussions of their theory, because they operate the theory with outdated values (very high detection ratios); only recent studies suggest much lower and realistic ratios.

So how are we supposed to extrapolate that the economists, who both intended for the 'accurate' high ratios to be ignored by the courts and were very likely ignorant of the recent studies of lower ratios, completely license the new implications/conclusions of their theory in light of the recent studies?

Furthermore, if we take a monotone approach to make A work, we run into problems with also (in my opinion) making D work. Maybe we just say that hey, the economists had a view which exhaustively defined reasonable penalties in a full reckoning of cost/benefit. So, if that full reckoning takes place and businesses turn belly-up, sucks to be the business but their full reckoning took place so therefore the economists must agree with what happened, as their given sufficient conditions for a just penalty have been checked off.

If you take that route, you just have to grant that the economists can agree with certain features/implications of their theory, even if they didn't consciously anticipate or intend them. So then, how would D be wrong? The whole point of including detection ratios in determining penalties is to make continued lawful action the less expensive action. Detection ratios do that by determining penalties with respect to the likelihood of the corporations recommitting a particular crime.

If economists don't agree that the likelihood of corporations recommitting a particular crime should be a main factor in determining the size of the penalty, then how would the passage show that economists agree that the possibility of the corporation going out of business is irrelevant to the penalty, since the mechanism by which the economists would license corporations going out of business just is the inclusion of sensitivity towards the likelihood of corporations recommitting the crime (detection ratios)?


I know it's long, thanks to anyone in advance for reading and responding. Don't be too harsh if I can't see the forest from the trees or something like that..
 Adam Tyson
PowerScore Staff
  • PowerScore Staff
  • Posts: 5153
  • Joined: Apr 14, 2011
|
#68472
That's a lot to digest, but let me see what I can do here!

First, we have to stick to what the passage says, and base our answer solely on that. The economists in the passage are given only one view - not a naive one and then a reformed, better one. That one view they hold is "the sole basis for determining the penalty should be the reckoning of cost and benefit: the penalty levied should exceed the profit that accrued to the corporation as a result of committing the crime."

It is the author, and not the economists, who goes on to say that their view is problematic because it requires that detection rates be high enough for it to make good sense, and that detection rates are not nearly high enough for that. We can't know if the economists ever gave any thought to detection rates, or if they overestimated them. All we know is that the author thinks their view is too simplistic, that it fails to address a key issue, and that it also overlooks the unintended consequence of harming workers.

So for this question, forget detection rates. Focus solely on what we know the economists said. If the sole basis for determining penalties should be that the penalty exceeds the profit from the crime, then nothing else matters. Going out of business, detection rates, the possibility of committing the crime again or the rate of them having been convicted in the past - all of it is out the window in their eyes. That's why we can and should select answer A - the text of the passage supports it in plain language.

Get the most out of your LSAT Prep Plus subscription.

Analyze and track your performance with our Testing and Analytics Package.